Where could point E be located on the coordinate plane? I’ll mark you as brainliest! Quadrant one, or Quadrant two, or Quadrant three or , Quadrant four , or Y axis, or X axis?

Where Could Point E Be Located On The Coordinate Plane? Ill Mark You As Brainliest! Quadrant One, Or

Answers

Answer 1

Answer:

Quadrant II and III

Step-by-step explanation:

Given

Point E

[tex]x < 0[/tex] i.e. negative x

and

[tex]y \ne 0[/tex]

Required: Determine the possible quadrant(s) of E

[tex]y \ne 0[/tex]. means that point E belongs to at least 1 quadrant. So, x-axis and y-axis are ruled out

Solving further:

[tex]x < 0[/tex]

The two quadrants where the x coordinates is negative are II and III.

Hence, point E belongs to quadrants II or quadrants III


Related Questions

During his career, Shaquille O'Neal attempted 22 three pointers. He made one of them. What is Shaq's 3-point percentage to the nearest tenth of a percent?

Answers

4.5% because 1/22 is 0.045 then move the decimal point 2 times for the percentage

5·(y-3)=3/5 (x-8)·5 write the equation

Answers

Answer:

This is the answer

Step-by-step explanation:

You needed to multiple 5 to both sides

Explain how to determine whether two quantities are proportional.


please help :)

Answers

Answer:

* Ratios are proportional if they represent the same relationship.

*If the reduced fractions are the same, your ratios are proportional.

*If two ratios are proportional is to write them as fractions and then reduce them.

Hope its helpful

Thank you

Jayvion took out a simple interest loan to pay for some new furniture. If the loan was for 5 years at 12% and he paid $960 interest, how much money did he borrow?

Answers

Answer:

$544.73

General Formulas and Concepts:

Pre-Algebra

Order of Operations: BPEMDAS

Brackets Parenthesis Exponents Multiplication Division Addition Subtraction Left to Right

Algebra I

Simple Interest Rate Formula: [tex]\displaystyle A = P(1 + r)^t[/tex]

A is final amountP is principle amountr is ratet is time

Step-by-step explanation:

Step 1: Define

A = 960

r = 0.12

t = 5

Step 2: Solve for P

Substitute in variables [Simple Interest Rate Formula]:                                [tex]\displaystyle 960 = P(1 + 0.12)^5[/tex][Interest] (Parenthesis) Add:                                                                            [tex]\displaystyle 960 = P(1.12)^5[/tex][Interest] Evaluate exponents:                                                                        [tex]\displaystyle 960 = P(1.76234)[/tex][Interest] [Division Property of Equality] Isolate P:                                        [tex]\displaystyle 544.73 = P[/tex][Interest] Rewrite:                                                                                            [tex]\displaystyle P = 544.73[/tex]

State the domain and range of both the function and the inverse function. f(x) = 8 x + 8 f -1(x) = Domain of f(x): Range of f(x): Domain of f -1(x): Range of f -1(x)

Answers

Answer:

[tex]f^{-1}(x) = \frac{x+8}{8}[/tex]

Domain of f(x): All real values.

Range of f(x): All real values.

Domain of f -1(x): All real values.

Range of f -1(x): All real values

Step-by-step explanation:

We are given the following function:

[tex]f(x) = 8x + 8[/tex]

Since it is a line, we have that both the domain is the range is the real set.

Inverse function:

The domain of the inverse function is the range of the original function, and the range of the inverse function is the domain of the original function.

Finding the inverse function:

Exchange x and y in the original function, and isolate y. So

Original

[tex]y = 8x + 8[/tex]

Exchanging:

[tex]x = 8y - 8[/tex]

Isolating y:

[tex]8y = x + 8[/tex]

[tex]y = \frac{x+8}{8}[/tex]

[tex]f^{-1}(x) = \frac{x+8}{8}[/tex]

Identify the percent of change as an increase or a decrease. 4/5 to 3/5 Increase Decrease Find the percent of change. Round to the nearest tenth of a percent if necessary.

Answers

The required percent of change from 4/5 to 3/5 is a decrease of 25%.

What is the percentage?

The percentage is the ratio of the composition of value to the overall composition of value multiplied by 100.

Here,
The fraction 4/5 is greater than the fraction 3/5, so the change from 4/5 to 3/5 represents a decrease.

To find the percent of change, we first need to find the amount of change. We can do this by subtracting the original value (4/5) from the new value (3/5),

3/5 - 4/5 = -1/5

The amount of change is -1/5. To express this as a percent, we can use the formula.,

Percent change = (amount of change / original value) * 100%

Substituting the values we have,

Percent change = (-1/5 / 4/5) * 100%

Percent change = -0.25 * 100%

Percent change = -25%

Therefore, the percent of change from 4/5 to 3/5 is a decrease of 25%.

Learn more about percentages here:

brainly.com/question/13450942

#SPJ

Describe how to modify a Tiling diagram

Answers

Do you mean triangle diagram ? Or like title floor

Quick help!!! Also pls show work!

Answers

Answer:

10a. No

10b. No

10c. Yes

10d. Yes

Step-by-step explanation:

10a. This equation does not because 3/4 x 8 = 6 and not 9.

10b. This equation doesn’t because 3x8 does not equal 28.

10c. This equation does because 5 x 8 = 40 which means y = 8.

10d. This equation does because 8/2 = 4. Fractions are just division problems and 8 divided by 2 equals 4, meaning y = 8 was n this equation.

the west valley senior held a fundraiser in which they sold tshirts and hoodies. There were 85 more hoodies sold than tshirts. Each t-shirt sold for 14.50 and each hoodie sold for 19.75. The team raised a total of 6,028.50 for the sale. Write and solve a system of equations to determine the number of tshirts and hoodies sold. ​

Answers

What do u mean can u explain it better thanks you very much

what’s four times a number (algebra wise)

Answers

Answer:

4x

Step-by-step explanation:

Sam and Pam's pennies add up to 22. Same has 1 more penny than twice Pam's pennies. How many pennies do Sam and Pam have?

Answers

Answer:

pam has x = 7 pennies

sam has 2x + 1 = 14 + 1 = 15 pennies

Step-by-step explanation:

let's take the number of pennies that pam owns be x

sam's pennies = 2× ( pam's pennies ) + 1

=》2x + 1

if the total pennies owned by both = 22

then,

x + 2x +1 = 22

3x = 22 - 1

x = 21 ÷ 3

x = 7

so, pam has x = 7 pennies

sam has 2x + 1 = 14 + 1 = 15 pennies

Natalie wrote that 773.15-200.4=3,531. What mistake did she make?

Answers

Answer:

Well her mistake is she didn't subtract correctly

Step-by-step explanation:

Her answer is supoose to be 572.75

Answer:

She lined the decimals wrong (common mistake)

Step-by-step explanation:

The correct answer is 572.75

A bag contains 10 coloured balls. 5 of them are red and 3 of them are blue. A ball is taken from the bag at random, then replaced. A second ball is then selected at random. Find the probability that the first ball is red and the second ball is blue.

Answers

The probability that the first ball drawn is red and the second ball drawn is blue is 3/20.

The probability that the first ball drawn is red and the second ball drawn is blue, we need to multiply the probability of drawing a red ball on the first draw by the probability of drawing a blue ball on the second draw.

Given that there are 10 balls in total, with 5 red balls and 3 blue balls, the probability of drawing a red ball on the first draw is 5/10 = 1/2.

Since the ball is replaced after the first draw, the total number of balls remains the same for the second draw.

The probability of drawing a blue ball on the second draw is also 3/10.

To find the probability of both events occurring (the first ball being red and the second ball being blue), we multiply the probabilities:

Probability = (Probability of drawing a red ball) × (Probability of drawing a blue ball)

= (1/2) × (3/10)

= 3/20

For similar questions on probability

https://brainly.com/question/25839839

#SPJ8

the temperature fell 16 degrees fahrenheit from 50 to 34 degrees in 10 hours. How much would the temperature fall over 15 hours if the pattern continued?

Answers

Answer:

[tex]x = 24\ degrees[/tex]

Step-by-step explanation:

Given

16 degrees = 10 hours

Required

Determine the change over 16 hours

Let the change be represented with x,

So, we have:

16 degrees = 10 hours

x = 15 hours

Cross Multiply:

[tex]x * 10\ hours = 16\ degrees * 15\ hours[/tex]

[tex]x * 10 = 16\ degrees * 15[/tex]

Divide through by 10

[tex]x = \frac{16\ degrees * 15}{10}[/tex]

[tex]x = 16\ degrees * 1.5[/tex]

[tex]x = 24\ degrees[/tex]

For expressions a-e, select Yes or No to indicate whether each expression is equivalent
to 3(x + 2)
a. 3x + 2
Yes
No
b. 312 + x)
Yes
No
Yes
c. 3x + 2x
No
d. x + 2x + 2 + 4
Yes
No
Yes
No
e. x + x + x + 1 + 1 + 1 + 1 + 1 + 1

Answers

Answer:

a. 3x +2

Step-by-step explanation:

I hope DIS is help you

Answer:

Answer 1:

no

Answer 2:

yes

Answer 3:

no

Answer 4:

yes

Answer 5:

yes

Step-by-step explanation:

Mary’s annual salary is $38,500 plus 22% in employee benefits. Her yearly job expenses are: uniforms, $500; transportation and parking, $5,350; and tools and certifications, $1,200. What are her net job benefits?

Answers

Answer: $39,920

Step-by-step explanation:

Her net job benefits would be the benefits and salary she receives less the expenses she incurs in relation to the job:

= Salary + Benefits - Uniforms - Transport - Tools

= 38,500 + (22% * 38,500) - 500 - 5,350 - 1,200

= ‭46,970‬ - 7,050

= $39,920

PLEASE SOMEONE HELP ME PLZZZ ASAP

Answers

Answer: C. -11x^4+x^3+7x^2

Step-by-step explanation:

Combine like terms like -7x^4 and -4x^4, 5x^2+2x^2= 7x^2

Each of students reported the number of movies they saw in the past year. Here is what they reported. 17,19,10,4,16,20​

Answers

Answer:

14.3

Step-by-step explanation:

What you would do is add all 6 numbers together. After finding the sum, divide by 6. The 6 represents the amount of numbers in the data set. So you would find 86/6 and get 14.3333. That would be rounded to 14.3. Hope this helped!

which set of number are right for this inequality x<14 plz help me i need help due in 20 min
A. 3,7,9,10,12
B. 15, 18 ,20, 21, 23
C. 17, 21, 24, 25, 31
D. 23, 27, 29, 30, 36

Answers

Answer:

it's A because all of those numbers are less than 14

Step-by-step explanation:

858 is divisible by 2,3,5 or 10?

Answers

Answer:

2

Step-by-step explanation:

If it ends in an even number, it is divisible by an even number

Hope this helps!!

Answer:

Step-by-step explanation:

858/2 = 429 - divisible

858/3 = 286 - divisible

858/5 = 171.6 - not divisible

858/10 = 85.8 - not divisible

what is 9(u-3) to remove parentheses.

Answers

Answer:

Distribute the 9 among the values inside the parentheses.

So it would be: 9u - 27

PLEASE ANSWER ASAP CORRECTLY FOR BRAINLEST!!!!!!!!!!!!!!!!!!!!!!!!!!!!!!!!!!!!!!!!!!!!!!!!!!!!!!!!!

Answers

Answer:

40,009.88

Step-by-step explanation:

The circumference of a circle can be found by multiplying pi ( π = 3.14 ) by the diameter of the circle.

were given the radius, r=6371

diameter = r x 2

6371 x 2 = 12742

12742 = diameter

12,742 x 3.14 = 40,009.88

Answer:

40,030.173592041 km

Step-by-step explanation:

circumference of a circle = 2πr

=2 × π × r

=2 × π × (6371)

=40,030.173592041

10 POINTS (ignore the bottom and top questions pls lol)

Answers

112 ft because your suppose to add I think well depends if your trying to multiply it.

Simplify: (2x + 8)(7x2 - 9x - 5)

Answers

Answer:

-18x^2 -54x + 72

Step-by-step explanation:

(2x+8)(7x2-9x-5) simplifies to -18x2−54x+72

The number of two digits which is such that, the
sum of the digits is 12 and the digit in the unit
place is three times the digit in tens place is:
a. 39
b. 40
c. 41
d. 42​

Answers

Answer:

39

Step-by-step explanation:

one number = x

another = 3x

x+3x = 12

4x = 12

x = 12/4 = 3

units place = 3x = 9

tens place= x = 3

number = 39

Escribe V si la afirmación es verdadera y F si es falsa, de ser falsa justifica la elección.

A) Todos los números son racionales ( )
B)Todos los números naturales son racionales ( )
C) Ningún numero natural es racional ( )
D)Todos los números racionales son naturales ( )
E)Todos los números enteros son racionales ( )
F)Todos los números racionales son enteros ( )
G) Ningún numero entero es racional ( )
H)Todos los números decimales son racionales ( )​

Answers

Answer:

Step-by-step explanation:

Todo numero natural es entero?

222222+999999what is this answer?​

Answers

Answer:

888888

Step-by-step explanation:

Is the right answer

Answer:

1222221

Step-by-step explanation:

222222+999999

Start from the ones place, 2+9=11 (11)

Borrow 1, 2+9=11+1=12 (21)

Again, 2+9=11+1=12 (221)

Again, 2+9=11+1=12 (2221)

Again, 2+9=11+1=12 (22221)

And again, 2+9=11+1=12 (1222221)

Hope I helped! Good Luck!

Solve for x in 3(5x) = 246. Round to four decimal places

Answers

Answer:

x = 16.4

General Formulas and Concepts:

Pre-Algebra

Order of Operations: BPEMDAS

Brackets Parenthesis Exponents Multiplication Division Addition Subtraction Left to Right  

Equality Properties

Multiplication Property of Equality Division Property of Equality Addition Property of Equality Subtraction Property of Equality

Step-by-step explanation:

Step 1: Define

3(5x) = 246

Step 2: Solve for x

Multiply:                                                                                                             15x = 246[Division Property of Equality] Divide 15 on both sides:                                x = 82/5Convert [Divide]:                                                                                               x = 16.4

Answer:

16.4

Step-by-step explanation:

3(5x)=246

15x=246

divide by 15

16.4

Cindy has a 4-shelf bookcase in which she wants to place baskets. Each basket takes up 1/4 of the shelf. How many baskets can Cindy place on the 4 shelves?

Answers

I believe it would be 16. If each basket takes up 1/4 of 1 shelf then 4 baskets would fit per shelf. 4 x 4 = 16

Answer:

16

Step-by-step explanation:

One basket takes (1/4)th of the shelf.

So, in one shell, Cindy can place 4 baskets

Total number of baskets = 4 *4 = 16

What is the equation of a horizontal line that passes through the point (-3, 4)?
A. y=3 B. y=-3 C. y=4 D.y=-4

Answers

Answer: C

Step-by-step explanation:

Since the points are ( -3,4 ) it passes through 4 in the y axis.

Meaning it will pass through 4 which is C.

Answer:

y = 4

Step-by-step explanation:

Other Questions
Please help me will give brainlest to correct answer and simple/small explanation Create a question "three branches of government" ?(do not copy from other website lol) 3. Some organisms have traits that helpthem survive in certain situations,while others of the same species donot. These differences happen bydue to genetic variation throughsexual reproduction,chanceplanning Help me please people keep taking y points and not answering my questions Solve each of the systems below by substitutiony = -3/2x (fraction)3x + 2y = -4 Si una matriz tiene una fila o columna de ceros su determinante es:A mismo.B. B. dobleC. El opuesto What is an equation of the line that passes through the points (8, 5)and (-6, 5)? The journal entry to record the purchase of materials on account is a(n) What social opportunities Does London provide Plz hurry its urgent Thank you In the passage An Occurrence at Owl Creek Bridge, Why does the author say "It did not appear to be the duty of these two men toknow what was occurring at the center of the bridge; they merely blockaded the two ends of the foot planking that traversed it?A)To emphasize the fact that the soldiers did not know what was about tohappen.B)To emphasize the fact that they respect the soldiers who are about to die.c)To emphasize the fact that they are simply following the orders of theircommanders.D)To emphasize the fact that the men knew they might be put to death next. What are the values of m and n in each right triangle Find the circumference of the circle. please hurry can you please help me with my work write a letter to your friend who is studying in class 10 and wishing her good luck in the board examination Helpppppppppppppopppppp How do languages share space? 1. What does the dermal tissue system do? plz help answer if you know the answer 8x-5 = 9+7x What is the solution please show work if you can What's the smallest possible perimeter of a rectangle with an area of 35 cm? (36 small squares 1 cm by 1 cm)